Subspace topology and product topology












2












$begingroup$


Let $tau$ be a topology on a topological space $X times Y$ which is not a product topology. Consider the subspace topology $tau_X$ and $tau_Y$ induced by the topology $tau$. My question is would the product topology $tau_X times tau_Y$ on $X times Y$ coincide with the topology $tau$.



I tried proving this by showing open sets in one topology is contained in other. I think I can show that $tau subset tau_X times tau_Y$ but I am not able to show the other way round.










share|cite|improve this question









$endgroup$








  • 2




    $begingroup$
    $tau$ is not a product topology and $tau_Xtimestau_Y$ is a product topology. This is enough to conclude that the topologies do not coincide.
    $endgroup$
    – drhab
    Dec 7 '18 at 15:59


















2












$begingroup$


Let $tau$ be a topology on a topological space $X times Y$ which is not a product topology. Consider the subspace topology $tau_X$ and $tau_Y$ induced by the topology $tau$. My question is would the product topology $tau_X times tau_Y$ on $X times Y$ coincide with the topology $tau$.



I tried proving this by showing open sets in one topology is contained in other. I think I can show that $tau subset tau_X times tau_Y$ but I am not able to show the other way round.










share|cite|improve this question









$endgroup$








  • 2




    $begingroup$
    $tau$ is not a product topology and $tau_Xtimestau_Y$ is a product topology. This is enough to conclude that the topologies do not coincide.
    $endgroup$
    – drhab
    Dec 7 '18 at 15:59
















2












2








2





$begingroup$


Let $tau$ be a topology on a topological space $X times Y$ which is not a product topology. Consider the subspace topology $tau_X$ and $tau_Y$ induced by the topology $tau$. My question is would the product topology $tau_X times tau_Y$ on $X times Y$ coincide with the topology $tau$.



I tried proving this by showing open sets in one topology is contained in other. I think I can show that $tau subset tau_X times tau_Y$ but I am not able to show the other way round.










share|cite|improve this question









$endgroup$




Let $tau$ be a topology on a topological space $X times Y$ which is not a product topology. Consider the subspace topology $tau_X$ and $tau_Y$ induced by the topology $tau$. My question is would the product topology $tau_X times tau_Y$ on $X times Y$ coincide with the topology $tau$.



I tried proving this by showing open sets in one topology is contained in other. I think I can show that $tau subset tau_X times tau_Y$ but I am not able to show the other way round.







general-topology






share|cite|improve this question













share|cite|improve this question











share|cite|improve this question




share|cite|improve this question










asked Dec 7 '18 at 15:51









ShreyShrey

132




132








  • 2




    $begingroup$
    $tau$ is not a product topology and $tau_Xtimestau_Y$ is a product topology. This is enough to conclude that the topologies do not coincide.
    $endgroup$
    – drhab
    Dec 7 '18 at 15:59
















  • 2




    $begingroup$
    $tau$ is not a product topology and $tau_Xtimestau_Y$ is a product topology. This is enough to conclude that the topologies do not coincide.
    $endgroup$
    – drhab
    Dec 7 '18 at 15:59










2




2




$begingroup$
$tau$ is not a product topology and $tau_Xtimestau_Y$ is a product topology. This is enough to conclude that the topologies do not coincide.
$endgroup$
– drhab
Dec 7 '18 at 15:59






$begingroup$
$tau$ is not a product topology and $tau_Xtimestau_Y$ is a product topology. This is enough to conclude that the topologies do not coincide.
$endgroup$
– drhab
Dec 7 '18 at 15:59












1 Answer
1






active

oldest

votes


















3












$begingroup$

You are considering subspace topologies on what subsets? $X$ and $Y$ are not subsets of their cartesian product; you could embed them via some injection, for which on the other hand there is no canonical choice (and would not even exist in the extreme case when only one of your two sets is nonempty).






share|cite|improve this answer









$endgroup$













    Your Answer





    StackExchange.ifUsing("editor", function () {
    return StackExchange.using("mathjaxEditing", function () {
    StackExchange.MarkdownEditor.creationCallbacks.add(function (editor, postfix) {
    StackExchange.mathjaxEditing.prepareWmdForMathJax(editor, postfix, [["$", "$"], ["\\(","\\)"]]);
    });
    });
    }, "mathjax-editing");

    StackExchange.ready(function() {
    var channelOptions = {
    tags: "".split(" "),
    id: "69"
    };
    initTagRenderer("".split(" "), "".split(" "), channelOptions);

    StackExchange.using("externalEditor", function() {
    // Have to fire editor after snippets, if snippets enabled
    if (StackExchange.settings.snippets.snippetsEnabled) {
    StackExchange.using("snippets", function() {
    createEditor();
    });
    }
    else {
    createEditor();
    }
    });

    function createEditor() {
    StackExchange.prepareEditor({
    heartbeatType: 'answer',
    autoActivateHeartbeat: false,
    convertImagesToLinks: true,
    noModals: true,
    showLowRepImageUploadWarning: true,
    reputationToPostImages: 10,
    bindNavPrevention: true,
    postfix: "",
    imageUploader: {
    brandingHtml: "Powered by u003ca class="icon-imgur-white" href="https://imgur.com/"u003eu003c/au003e",
    contentPolicyHtml: "User contributions licensed under u003ca href="https://creativecommons.org/licenses/by-sa/3.0/"u003ecc by-sa 3.0 with attribution requiredu003c/au003e u003ca href="https://stackoverflow.com/legal/content-policy"u003e(content policy)u003c/au003e",
    allowUrls: true
    },
    noCode: true, onDemand: true,
    discardSelector: ".discard-answer"
    ,immediatelyShowMarkdownHelp:true
    });


    }
    });














    draft saved

    draft discarded


















    StackExchange.ready(
    function () {
    StackExchange.openid.initPostLogin('.new-post-login', 'https%3a%2f%2fmath.stackexchange.com%2fquestions%2f3030041%2fsubspace-topology-and-product-topology%23new-answer', 'question_page');
    }
    );

    Post as a guest















    Required, but never shown

























    1 Answer
    1






    active

    oldest

    votes








    1 Answer
    1






    active

    oldest

    votes









    active

    oldest

    votes






    active

    oldest

    votes









    3












    $begingroup$

    You are considering subspace topologies on what subsets? $X$ and $Y$ are not subsets of their cartesian product; you could embed them via some injection, for which on the other hand there is no canonical choice (and would not even exist in the extreme case when only one of your two sets is nonempty).






    share|cite|improve this answer









    $endgroup$


















      3












      $begingroup$

      You are considering subspace topologies on what subsets? $X$ and $Y$ are not subsets of their cartesian product; you could embed them via some injection, for which on the other hand there is no canonical choice (and would not even exist in the extreme case when only one of your two sets is nonempty).






      share|cite|improve this answer









      $endgroup$
















        3












        3








        3





        $begingroup$

        You are considering subspace topologies on what subsets? $X$ and $Y$ are not subsets of their cartesian product; you could embed them via some injection, for which on the other hand there is no canonical choice (and would not even exist in the extreme case when only one of your two sets is nonempty).






        share|cite|improve this answer









        $endgroup$



        You are considering subspace topologies on what subsets? $X$ and $Y$ are not subsets of their cartesian product; you could embed them via some injection, for which on the other hand there is no canonical choice (and would not even exist in the extreme case when only one of your two sets is nonempty).







        share|cite|improve this answer












        share|cite|improve this answer



        share|cite|improve this answer










        answered Dec 7 '18 at 15:58









        ΑΘΩΑΘΩ

        2563




        2563






























            draft saved

            draft discarded




















































            Thanks for contributing an answer to Mathematics Stack Exchange!


            • Please be sure to answer the question. Provide details and share your research!

            But avoid



            • Asking for help, clarification, or responding to other answers.

            • Making statements based on opinion; back them up with references or personal experience.


            Use MathJax to format equations. MathJax reference.


            To learn more, see our tips on writing great answers.




            draft saved


            draft discarded














            StackExchange.ready(
            function () {
            StackExchange.openid.initPostLogin('.new-post-login', 'https%3a%2f%2fmath.stackexchange.com%2fquestions%2f3030041%2fsubspace-topology-and-product-topology%23new-answer', 'question_page');
            }
            );

            Post as a guest















            Required, but never shown





















































            Required, but never shown














            Required, but never shown












            Required, but never shown







            Required, but never shown

































            Required, but never shown














            Required, but never shown












            Required, but never shown







            Required, but never shown







            Popular posts from this blog

            Plaza Victoria

            In PowerPoint, is there a keyboard shortcut for bulleted / numbered list?

            How to put 3 figures in Latex with 2 figures side by side and 1 below these side by side images but in...